Game Theory ( 2 player zero-sum game)

Click For Summary
SUMMARY

This discussion focuses on the application of minimax strategies in two-player zero-sum games, specifically analyzing equilibrium solutions in modified game matrices. The original game matrix A is defined as A = [[a, b], [c, d]], with an equilibrium solution at (e1; e1). The modified matrix A0 is presented as A0 = [[a + K, b + K], [c + K, d + K]], where K is a constant. The discussion confirms that (e1; e1) remains an equilibrium solution in the new matrix, and calculations involving the matrices A1 and A2 demonstrate the relationship between equilibrium solutions and constant adjustments.

PREREQUISITES
  • Understanding of minimax theorem in game theory
  • Familiarity with zero-sum game concepts
  • Basic knowledge of matrix operations
  • Concept of Nash equilibrium in two-player games
NEXT STEPS
  • Study the minimax theorem in detail
  • Explore Nash equilibrium concepts in multi-player games
  • Learn about matrix transformations in game theory
  • Investigate applications of zero-sum games in economics
USEFUL FOR

This discussion is beneficial for students of game theory, mathematicians focusing on strategic decision-making, and anyone interested in the theoretical foundations of competitive scenarios.

ruby_duby
Messages
46
Reaction score
0
Hi

I have just started a game theory module and have been given the following exercise. Can anyone help me with the following question. I have a feeling I can use a minimax argument to answer this but I am not sure how to go about this.

Suppose a two-player zero-sum game has the following matrix:
A =

a b
c d

Suppose that (e1; e1) is an equilibrium solution. Suppose K a member of the ℝ is a constant. Show for the new game matrix:
A0 =

a + K b + K
c + K d + K

that (e1; e1) is still the equilibrium solution.
 
Physics news on Phys.org
if i let Suppose a two-player zero-sum game has the following matrix:

A1 =

a b
c d

then calculate e2TA1e1 = c

Then if I let A2 =

a+k b+k
c+k d+k

then e2TA2e1 = c+k

but I have no idea if this is right or what to do next
 
Question: A clock's minute hand has length 4 and its hour hand has length 3. What is the distance between the tips at the moment when it is increasing most rapidly?(Putnam Exam Question) Answer: Making assumption that both the hands moves at constant angular velocities, the answer is ## \sqrt{7} .## But don't you think this assumption is somewhat doubtful and wrong?

Similar threads

  • · Replies 4 ·
Replies
4
Views
4K
  • · Replies 2 ·
Replies
2
Views
3K
  • · Replies 3 ·
Replies
3
Views
4K
  • · Replies 29 ·
Replies
29
Views
2K
  • · Replies 26 ·
Replies
26
Views
2K
Replies
9
Views
2K
  • · Replies 2 ·
Replies
2
Views
2K
  • · Replies 4 ·
Replies
4
Views
3K
  • · Replies 11 ·
Replies
11
Views
4K
  • · Replies 6 ·
Replies
6
Views
2K